Difference between revisions of "2008 iTest Problems/Problem 4"

m (Solution)
m (Solution)
 
Line 6: Line 6:
  
 
We know that any prime number, excluding <math>2</math>, is congruent to <math>1 \pmod 2</math>. Thus, if both of the primes are not <math>2</math>, their difference would be congruent to <math>0 \pmod 2</math>. Because <math>11 \equiv 1 \pmod 2</math>, one of the primes must be <math>2</math>. It follows that the other prime must then be <math>13</math>. Therefore, the sum of the two is <math>13+2=\boxed{15}</math>.
 
We know that any prime number, excluding <math>2</math>, is congruent to <math>1 \pmod 2</math>. Thus, if both of the primes are not <math>2</math>, their difference would be congruent to <math>0 \pmod 2</math>. Because <math>11 \equiv 1 \pmod 2</math>, one of the primes must be <math>2</math>. It follows that the other prime must then be <math>13</math>. Therefore, the sum of the two is <math>13+2=\boxed{15}</math>.
 +
 +
== Solution 2 ==
 +
 +
Since the difference is <math>11</math>, one number must be even for the difference to be even. <math>2</math> is the only even prime number, and therefore is one of the two numbers. The other number is <math>2 + 11 = 13</math>, and their sum: <math>13 + 2 = \boxed{15}</math>.
  
 
==See Also==
 
==See Also==

Latest revision as of 17:43, 24 November 2018

Problem

The difference between two prime numbers is $11$. Find their sum.

Solution

We know that any prime number, excluding $2$, is congruent to $1 \pmod 2$. Thus, if both of the primes are not $2$, their difference would be congruent to $0 \pmod 2$. Because $11 \equiv 1 \pmod 2$, one of the primes must be $2$. It follows that the other prime must then be $13$. Therefore, the sum of the two is $13+2=\boxed{15}$.

Solution 2

Since the difference is $11$, one number must be even for the difference to be even. $2$ is the only even prime number, and therefore is one of the two numbers. The other number is $2 + 11 = 13$, and their sum: $13 + 2 = \boxed{15}$.

See Also

2008 iTest (Problems)
Preceded by:
Problem 3
Followed by:
Problem 5
1 2 3 4 5 6 7 8 9 10 11 12 13 14 15 16 17 18 19 20 21 22 23 24 25 26 27 28 29 30 31 32 33 34 35 36 37 38 39 40 41 42 43 44 45 46 47 48 49 50 51 52 53 54 55 56 57 58 59 60 61 62 63 64 65 66 67 68 69 70 71 72 73 74 75 76 77 78 79 80 81 82 83 84 85 86 87 88 89 90 91 92 93 94 95 96 97 98 99 100